Akademisyenler öncülüğünde matematik/fizik/bilgisayar bilimleri soru cevap platformu
1 beğenilme 0 beğenilmeme
499 kez görüntülendi


Lisans Matematik kategorisinde (621 puan) tarafından  | 499 kez görüntülendi

Eğer fonkisyon (a,b) aralığında konveks ise $f(a_1x_1+a_2x_2+....+a_nf(x_n))\leq a_1f(x_1)+a_2f(x_2)+....a_nf(x_n)$  $a_1+a_2+...a_n=1$ 

1 cevap

0 beğenilme 0 beğenilmeme

direk olarak Jensen eşitsizliği

(1.8k puan) tarafından 

peki ispati nasil?

sorduğunuz sorunun ispatı ise,soruda verilen eşitsizlik zaten Jensen eşitsizliğinin iki terim için yazılışı. ama soruya teoremin genel formunu ekleyebilirim. eğer jensen eşisizliğinin kanıtını istiyorsanız onu yazmak biraz uzun sürer.
20,217 soru
21,748 cevap
73,344 yorum
1,968,035 kullanıcı